Eficiencia del motor Stirling y teorema de Carnot

Quiero calcular la eficiencia de este ciclo de Stirling para un gas ideal. pags V = norte R T

tomado de Nolting.  Grundkurs Theoretische Physik 4. Spezielle Relativitätstheorie und Thermodynamik

El trabajo mecánico es

Δ W 12 = V 1 V 2 pags ( V ) d V = norte R T 2 en V 2 V 1 Δ W 23 = Δ W 41 = 0 Δ W 34 = norte R T 1 en V 1 V 2
En las curvas isotérmicas el cambio en la energía interna Δ tu = Δ W + Δ q es cero
Δ q 12 = Δ W 12 > 0 Δ q 34 = Δ W 34 < 0
En las curvas isocóricas (isovolumétricas) las cantidades de calor son
Δ q 23 = C V ( T 1 T 2 ) < 0 Δ q 41 = C V ( T 2 T 1 ) > 0
La eficiencia es entonces
η = Δ W Δ q
Δ q es el calor de entrada, es decir, la suma de todas las cantidades de calor > 0 :
Δ q = q 12 + q 41 = norte R T 2 en V 2 V 1 + C V ( T 2 + T 1 )
Δ W es el trabajo mecánico total:
Δ W = W 12 + Δ W 34 = norte R ( T 2 T 1 ) en V 2 V 1

Entonces finalmente la eficiencia es

η = T 2 T 1 T 2 + C V ( T 2 T 1 ) norte R en V 2 / V 1 < η C .
Es menor que la eficiencia del ciclo de Carnot. Pero debería ser igual si todos los procesos se realizan de forma reversible.

Los cálculos están tomados de un libro de texto (Nolting: Grundkurs Theoretische Physik 4) que en realidad señala este problema como una pregunta al lector. Mi única explicación es que este proceso no es reversible, pero no sé cómo decirlo sin ver cómo se realizan los procesos isotérmicos e isocóricos.

Entonces mis preguntas son:

  • ¿Es esto una contradicción con el teorema de Carnot de que la eficiencia η C = 1 T 1 / T 2 es lo mismo para todas las máquinas térmicas reversibles entre dos baños térmicos?
  • ¿Este ciclo es reversible?
  • ¿ Hay alguna manera de decir si un proceso es reversible o irreversible solo con una figura como la de arriba?
Estás calculando mal la eficiencia. Para un ciclo de motor arbitrario, tiene q H = q L + W , y mi = W q H . No estás dividiendo por tu calor de entrada.
Edité la pregunta para mayor claridad de lo que estoy calculando. El denominador de la eficiencia debe ser el calor que se pone en el motor. La forma en que estoy calculando el calor es la que es > 0 . ¿Estás de acuerdo?
Relacionado con la cuestión de la reversibilidad de las curvas en el espacio de estados termodinámicos: physics.stackexchange.com/questions/78405/…

Respuestas (5)

Respuesta renovada. 2017-07-01

No hay contradicción porque su análisis solo incluye lo que le sucede a la sustancia de trabajo gaseosa en el motor Stirling y descuida un componente crucial del motor llamado regenerador. Si el regenerador no se incluye como un componente del motor cuando realizamos el análisis de eficiencia, entonces no tenemos un dispositivo que califique como un motor térmico operando entre dos temperaturas y, por lo tanto, no deberíamos esperar que cumpla con el criterio de Carnot. Teorema como dije en la versión original de esta respuesta.

Sin embargo, si tomamos en cuenta correctamente el regenerador, encontramos que la eficiencia del motor es la eficiencia de Carnot.

Por supuesto, todo el análisis aquí es idealizado en el que asumimos, por ejemplo, que no hay pérdidas de energía debido a la fricción en los componentes del motor.

Detalles.

Un motor Stirling es más complejo que el PAGS - V diagrama dibujado en el enunciado de la pregunta parece indicar. Si reducimos conceptualmente el motor a su forma más simple, contiene dos componentes fundamentales:

  1. Una sustancia de trabajo gaseosa . Esta es la parte del motor cuyo estado termodinámico viaja a lo largo de la curva en el PAGS - V diagrama.
  2. Un regenerador . Esta parte del motor absorbe y almacena la energía cedida por la sustancia de trabajo gaseosa por transferencia de calor durante el proceso. 2 3 y luego devuelve esa misma energía a la sustancia de trabajo gaseosa durante el proceso 4 1 .

El punto crucial es que cuando se incluye el regenerador, no hay transferencia de calor neta hacia adentro o hacia afuera del motor durante los procesos. 2 3 y 4 1 . La energía que deja la sustancia de trabajo gaseosa durante el proceso. 2 3 por transferencia de calor se almacena en el regenerador, y ese calor luego se devuelve a la sustancia de trabajo durante el proceso 4 1 . No se transfiere calor entre el motor y su entorno durante estas etapas del ciclo.

De ello se deduce que el único calor transferido al motor como un todo se transfiere durante 1 2 . Esto califica al dispositivo como un motor térmico (consulte la respuesta anterior a continuación) y la eficiencia del motor se calcula como la relación entre la producción neta de trabajo dividida por la entrada de calor en el proceso. 1 2 . Esto produce la eficiencia de Carnot como debería.

Mi respuesta original afirmaba que el ciclo dibujado no representa la operación de un motor térmico que opera entre dos temperaturas, pero estaba descuidando el regenerador, y creo que esto es lo que hiciste implícitamente en el cálculo que también realizaste originalmente, y esto produjo la eficiencia incorrecta.

Respuesta original, incompleta.

No hay contradicción. El ciclo de Stirling que dibujó arriba es reversible pero no opera entre dos depósitos a temperaturas fijas T 1 y T 2 . Las partes isovolumétricas del ciclo operan a temperaturas que cambian continuamente (piense en la ley de los gases ideales).

Adición antigua. Tenga en cuenta que en termodinámica, se dice que una máquina térmica opera (o trabaja) entre (dos depósitos a) temperaturas T 1 y T 2 siempre que todo el calor que absorba o ceda lo haga a una de esas dos temperaturas.

Para dar crédito a esta definición (que está esencialmente implícita en la mayoría de las discusiones sobre motores térmicos que he visto), he aquí una cita del texto de termodinámica de Fermi:

En la sección anterior describimos una máquina cíclica reversible, la máquina de Carnot, que realiza una cantidad de trabajo L durante cada uno de sus ciclos absorbiendo una cantidad de calor q 2 de una fuente a temperatura t 2 y entregando una cantidad de calor q 1 a una fuente a la temperatura más baja t 1 . Diremos que tal motor trabaja entre las temperaturas t 1 y t 2 .

Entonces creo que no entendí lo que realmente significa operar entre dos depósitos a temperatura fija. El ciclo de Carnot consiste en adiabático e isotérmico. cambia la temperatura pero no se transfiere calor. Entonces, ¿"operar" básicamente significa transferencia de calor? También dijiste que el ciclo es reversible. ¿Son todas las curvas que se pueden dibujar dentro de la pags - V -plano reversible?
@frankundfrei Sí; "operar entre dos temperaturas" en este contexto significa que todas las transferencias de calor que ocurren durante el ciclo ocurren a una u otra temperatura. En cuanto a la cuestión de la reversibilidad, no estoy seguro de que sea apropiado llamar reversible o irreversible a una curva en el espacio de estados termodinámicos. Creo que debemos responder una pregunta más matizada como: cuando realizamos un proceso físico idealizado, y sus estados sucesivos pueden aproximarse bien mediante una curva continua en el espacio de estado, ¿se puede realizar ese proceso a la inversa?
¿Podría agregar esta definición de "operación" en su respuesta? No he encontrado una definición precisa ni en la conferencia a la que asistí ni en los libros de texto que uso, por lo que también podría ser útil para otras personas. ¡Gracias por tu ayuda!
@frankundfrei Claro. Hice una edición.
@joshphysics: la prueba del teorema de Carnot, es decir, un ciclo reversible tiene la misma eficiencia que un ciclo de Carnot, no asume que el calor se transfiere al gas de forma isotérmica, por lo que no entiendo por qué la transferencia de calor isovolumétricamente cambia algo. Si el fluido de trabajo entra en contacto térmico con una fuente caliente, podemos calentar el fluido a volumen constante hasta que alcance la temperatura de la fuente caliente. No necesitamos que cambie la temperatura de la fuente caliente, ¿verdad?
@JoshuaBenabou Ha pasado mucho tiempo desde que pensé en estas cosas, pero soy escéptico con respecto a su afirmación sobre la generalidad del teorema de Carnot. Por lo que puedo decir, el teorema generalmente compara todos los motores térmicos que operan entre depósitos fijos de ciertas temperaturas: en.wikipedia.org/wiki/Carnot%27s_theorem_(thermodynamics)
@joshphysics: de hecho, el teorema habla de motores térmicos que operan entre dos depósitos fijos de temperaturas fijas. Personalmente, creo que su respuesta no es completamente correcta porque el OP calculó la eficiencia del ciclo de Stirling asumiendo que opera entre solo dos depósitos de temperaturas fijas. Por lo tanto, no es reversible (ya que la transferencia de calor a través de una diferencia de temperatura no infinitesimal crea entropía). Por lo tanto, tiene sentido que la eficiencia del ciclo de Stirling que encontró sea menor que la de un ciclo de Carnot que opera entre las mismas dos temperaturas.
@JoshuaBenabou He estado pensando bastante en esto gracias a sus comentarios, y he editado mi respuesta en consecuencia. En particular, no creo que un motor Stirling ideal sea irreversible como usted indica. La transferencia de calor entre la sustancia de trabajo del motor y el regenerador es, idealmente, reversible. Además, si el motor se analiza adecuadamente (ver respuesta renovada), la eficiencia de Carnot se obtiene como se desea. Sin embargo, en términos generales, tenía razón en ser escéptico: la respuesta original era muy incompleta y engañosa.

El ciclo de Stirling como lo describe no es reversible. La transferencia de calor desde depósitos térmicos a lo largo de los caminos 4->1 y 2->3 no es un proceso reversible, porque el calor se transfiere entre dos objetos a diferentes temperaturas. Para revertir el proceso, necesitarías transferir calor espontáneamente de un depósito más frío a uno más caliente, lo que viola la segunda ley de la termodinámica.

Los motores Stirling a menudo se describen como reversibles, pero esto requiere un tipo especial de proceso. Observe que el calor transferido al motor a lo largo de 4->1 es el mismo que el calor transferido fuera del motor a lo largo de 2->3 y que 4->1 y 2->3 operan entre las mismas dos temperaturas. Por lo tanto, se puede construir un motor Stirling eficiente de Carnot si el calor se transfiere isotérmicamente dentro del motor a lo largo de estos caminos. Esto se logra con un "regenerador", una masa térmica que almacena la energía liberada en 2->3 y la devuelve al gas por el camino 4->1. Puede ver que el regenerador tiene que variar continuamente en temperatura entre T2 y T1 e intercambiar calor isotérmicamente con el gas a medida que pasa.

Tenga en cuenta que todos los motores reversibles deben funcionar con la misma eficiencia. Esto se sigue de las definiciones de eficiencia y entropía. Un motor reversible opera con 0 cambio de entropía. Δ S = q h T h + q C T C , asi que Δ S = 0 implica q h T h = q C T C o eficiencia = q h q C q h = T h T C T h

No entiendo muy bien el "regenerador", pero creo que su argumento sobre la irreversibilidad de las rutas 4-> 1 y 2-> 3 no es correcto, también Δ S = 0 en el ciclo
@richard: debe considerar el cambio de entropía de todo el mundo, no solo del fluido de trabajo; cualquier ciclo cerrado tendrá un cambio de entropía de 0 para el fluido de trabajo; solo los motores reversibles tendrán un cambio de entropía de 0 para el fluido de trabajo y los depósitos térmicos.
sí, eso es lo que estoy diciendo si alguna parte del ciclo fuera irreversible, entonces el cambio de entropía del fluido de trabajo y también del mundo entero sería distinto de cero.
@Ricardo. para un gas ideal con un número fijo de partículas, cada punto en el diagrama P,V tiene una entropía específica (puede calcular esto, por ejemplo, a partir de la ecuación de sackur-tetrode). Por lo tanto, cualquier ciclo cerrado tiene un cambio de 0 en la entropía del fluido de trabajo. ¿Está afirmando que todos los ciclos cerrados son reversibles? Además, ciertos procesos reversibles, como la expansión isotérmica, cambian la entropía del fluido de trabajo. Debe considerar el cambio total en la entropía del fluido de trabajo y los depósitos juntos.
No, no estoy afirmando que todo ciclo cerrado sea reversible. Estoy diciendo que el ciclo en la imagen de arriba es reversible. Todas las partes son reversibles incluyendo 4-1 y 2-3.
@richard: En el camino 4-1, la entropía del gas aumenta. Si el calor fluye desde un depósito a la temperatura T1, la entropía del depósito disminuye, pero disminuye menos de lo que aumenta la entropía del gas. El cambio neto en la entropía es positivo y el proceso es irreversible. La única forma de hacer que 4-1 sea reversible es si el calor se transfiere isotérmicamente desde un conjunto de depósitos a cada temperatura intermedia entre T4 y T1. Esta es la función del regenerador que no entiendes del todo.
Creo que los caminos 4-1 y 2-3 en la imagen de arriba son los mismos que explicaste: conjunto de depósitos a temperaturas intermedias.
@richard: Es una cuestión de dónde viene el calor en la etapa 4-1 y hacia dónde se dirige en la etapa 2-3. En un motor Stirling ideal, el calor se transfiere internamente al regenerador en 2-3 y regresa del regenerador en 4-1, sin pérdida ni ganancia hacia o desde el exterior. Por lo tanto, un cambio cero en la entropía en estas etapas. Los motores Stirling reales no hacen esto de manera 100% eficiente, pero el motor Stirling ideal tiene un cambio de entropía cero en estas etapas. El diagrama PV es el mismo en ambos casos, pero los diagramas de entropía versus T serían diferentes.

En un ciclo de Stirling ideal, los pasos isocóricos intercambian calor a través de una diferencia de temperatura infinitesimal, que es mantenida por el regenerador que tiene un gradiente continuo de temperatura entre los depósitos caliente y frío. Luego, el gas puede enfriarse o calentarse en consonancia con ese gradiente. Esta es la parte ideal del diseño que permite un cambio cero en la entropía durante las dos etapas isocóricas. Este calor simplemente se baraja de un lado a otro internamente y, por lo tanto, el único intercambio real con el exterior es a través del depósito caliente y afuera a través del frío. De ahí la eficiencia ideal. No estoy seguro de que sea correcto llamar isotérmico a lo que sucede en las etapas del regenerador. La temperatura está cambiando continuamente pero idealmente siempre a través de una diferencia infinitesimal. ¿Hay un término comúnmente usado para eso? Sin embargo,

He notado en mis búsquedas en internet sobre el tema de los motores Stirling que muchas fuentes confunden estas ideas. A menudo he visto análisis de eficiencia que ignoran por completo el efecto del regenerador. Posiblemente esto tenga que ver con el hecho de que los procesos isocóricos no suelen estar asociados con un cambio cero en la entropía, pero en el caso del motor Stirling hay un tipo muy especial de este proceso involucrado, utilizando un regenerador.

El motor Stirling ideal tiene la misma eficiencia que el ciclo de Carnot, pero su ventaja es que permite la construcción de motores reales que, aunque no puedan lograr etapas isocóricas isotérmicas perfectas y regeneradoras totalmente suaves, se acercan y son mucho más factible que la posibilidad de construir un práctico motor de Carnot.

Entonces, en realidad, los motores Stirling fabricados reales no logran la eficiencia Carnot ideal completa, pero muchos lo hacen mucho mejor que otros tipos de motores térmicos.

En conclusión, considerando el motor Stirling ideal:

(1) Se logra la máxima eficiencia ideal del motor de Carnot. (2) Tu cálculo no contradice esto porque sea incorrecto. Usted incluye el calor intercambiado en las etapas isocóricas como parte del costo, mientras que el único costo es la entrada de calor externa durante la carrera de potencia isotérmica. (3) Este ciclo es reversible ya que no hay cambio en la entropía durante las etapas isocóricas. (4) El diagrama por sí solo no es suficiente para mostrar esto, ya que también debemos saber que el regenerador ideal es lo que permite el tercer punto. Es decir, si quitas el regenerador el diagrama sigue siendo el mismo.

El problema viene de la novena ecuación. Tenga en cuenta que el calor que se transfiere durante los dos procesos 4-1 y 2-3 se cancelan entre sí. El calor Q41 se entrega al regenerador y luego es reabsorbido por el material de trabajo del sistema. Esta cantidad de calor "no" la proporciona el depósito caliente al sistema ni la absorbe el depósito frío, sino que, en cierto sentido, se transfiere "reversiblemente" entre dos partes del propio material de trabajo. Entonces, incluyendo el Heat Q41 en la novena ecuación. como una parte del calor que "se transfiere" del conservante caliente al sistema es el punto incorrecto en los cálculos anteriores que ha llevado al resultado incorrecto dado en la ecuación 11.

¡Bienvenido a Physics.SE! Sugiero lo siguiente: 1) ¡Toma el tour ! 2) Cuando vea buenas preguntas y respuestas, vótelas haciendo clic en los triángulos grises , porque la credibilidad del sistema se basa en la reputación ganada por los usuarios que comparten su conocimiento. 3) Si tienes una buena pregunta, ¡hazla! Solo recuerda si lo haces y obtienes una respuesta satisfactoria para aceptarla haciendo clic en la marca de verificación verde. Además, si tiene alguna pregunta, consulte las pautas del centro de ayuda .

Cualquier ciclo en el diagrama pv es reversible. Cuando resuelves para Q, tienes que integrar y el proceso de integración en sí implica dT, lo que significa que la diferencia de temperaturas es infinitesimalmente pequeña, lo que hace que el proceso sea reversible. La fórmula de eficiencia del motor Stirling que ha obtenido es correcta, excepto que el número de moles (n) debería haberse cancelado. La eficiencia del motor Stirling es menor que la de Carnot y eso está bien. Como uno de ustedes mencionó, no se puede comparar con Carnot, ya que el intercambio de calor en el ciclo de Carnot tiene lugar a dos temperaturas fijas, mientras que en el motor Stirling el intercambio de calor también tiene lugar a lo largo de los dos procesos de volumen constante donde las temperaturas varían. En el ciclo de Carnot no hay intercambio de calor a lo largo de las curvas adiabáticas a lo largo de las cuales cambia la temperatura. Espero que esto ayude.